Difference between revisions of "1984 AIME Problems/Problem 13"
m (→Solution 4) |
Bluelinfish (talk | contribs) m (→Solution 1) |
||
| Line 21: | Line 21: | ||
<center><p><math>\tan((a+b)+(c+d)) = \frac{\frac{1}{2}+\frac{1}{8}}{1-\frac{1}{16}} = \frac{2}{3}</math>.</p></center> | <center><p><math>\tan((a+b)+(c+d)) = \frac{\frac{1}{2}+\frac{1}{8}}{1-\frac{1}{16}} = \frac{2}{3}</math>.</p></center> | ||
| − | Thus our answer is <math>10\cdot\frac{3}{2}= | + | Thus our answer is <math>10\cdot\frac{3}{2}=\boxed{015}</math>. |
=== Solution 2 === | === Solution 2 === | ||
Revision as of 20:02, 15 December 2018
Problem
Find the value of
Contents
Solution
Solution 1
We know that
so we can repeatedly apply the addition formula,
. Let
,
,
, and
. We have
,
So
![]()
and
,
so
.
Thus our answer is
.
Solution 2
Apply the formula
repeatedly. Using it twice on the inside, the desired sum becomes
. This sum can then be tackled by taking the cotangent of both sides of the inverse cotangent addition formula shown at the beginning.
Solution 3
On the coordinate plane, let
,
,
,
,
,
,
,
,
, and
. We see that
,
,
, and
. The sum of these four angles forms the angle of triangle
, which has a cotangent of
, which must mean that
. So the answer is
.
Solution 4
Recall that
and that
. Then letting
and
, we are left with
Expanding
, we are left with
See also
| 1984 AIME (Problems • Answer Key • Resources) | ||
| Preceded by Problem 12 |
Followed by Problem 14 | |
| 1 • 2 • 3 • 4 • 5 • 6 • 7 • 8 • 9 • 10 • 11 • 12 • 13 • 14 • 15 | ||
| All AIME Problems and Solutions | ||